Autor Nachricht
TomS
BeitragVerfasst am: 13. Mai 2013 19:59    Titel:

_Reggid hat Folgendes geschrieben:
kurze google suche hat allgemein "Furry's theorem" ergeben. in diesem speziellen fall sind es eigentlich zwei diagramme (ausgehende photon linien "kreuzen" wegen ununterscheidbarkeit), die sich canceln.

Perfekt, das hatte ich mal gelesen und wieder vergessen.

Man kann sowohl mit der Parität angewandt auf die Matrixelemente, als auch explizit mittels Berechnung der Diagramme argumentieren. Danke!
adaddsa
BeitragVerfasst am: 13. Mai 2013 19:03    Titel:

_Reggid hat Folgendes geschrieben:

wenn du aber meinst, dass dieses diagramm mit drei photonen sicher 0 sein muss, dann wäre mir zuerst noch paritätserhaltung in der QED als argument eingefallen, aber eigentlich sollte das bei dem prozess auch kein problem machen, oder? photon hat parität (-1), d.h. mit einem relativen drehimpuls von 1 für die auslaufenden teilchen sollte das mit parität und auch drehimpulserhaltung (spin der auslaufenden photonen anitparallel) in ordnung gehen.[/latex]
Das wird doch gerade mit C-Parität nicht passen, wegen (-1)^2
_Reggid
BeitragVerfasst am: 13. Mai 2013 19:01    Titel:

kurze google suche hat allgemein "Furry's theorem" ergeben. in diesem speziellen fall sind es eigentlich zwei diagramme (ausgehende photon linien "kreuzen" wegen ununterscheidbarkeit), die sich canceln.
_Reggid
BeitragVerfasst am: 13. Mai 2013 18:46    Titel:

darf ich nochmal nachfragen, warum du weißt, dass dieser prozess nicht möglich ist? schließlich ist die photon-photon-streuung (also mit vier externen photonen) über einen fermion-loop ja auch möglich, wenn auch natürlich stark unterdrückt. dieses diagramm könnte ich ja auch für ein einlaufendes und drei auslaufende photonen verwenden, was ja für masselose teilchen mit enerige- und impulserhaltung vereinbar ist. woher kann man unterscheiden, dass der prozess nicht erlaubt, der prozess schon erlaubt ist? ich habe außerdem immer gelernt, dass die photonenzahl sowieso nicht erhalten ist, bzw. nicht genau defniert ist.

wenn du aber meinst, dass dieses diagramm mit drei photonen sicher 0 sein muss, dann wäre mir zuerst noch paritätserhaltung in der QED als argument eingefallen, aber eigentlich sollte das bei dem prozess auch kein problem machen, oder? photon hat parität (-1), d.h. mit einem relativen drehimpuls von 1 für die auslaufenden teilchen sollte das mit parität und auch drehimpulserhaltung (spin der auslaufenden photonen anitparallel) in ordnung gehen.[/latex]
adaddsa
BeitragVerfasst am: 13. Mai 2013 17:58    Titel:

Vielleicht wäre für dich interessant:
"General properties of the decay amplitudes for massless particles
G. Fiore, G. Modanese"
TomS
BeitragVerfasst am: 13. Mai 2013 16:13    Titel: Photonenzerfall verboten in QED

Es geht um den hypothetischen Zerfall



mit einem unbekannten Teilchen-Antiteilchen-Paar (X).

Man kann sich leicht überlegen, dass dieser Zerfall verboten ist, da Energie- und Impulserhaltung nicht gleichzeitig gelten können, es sei denn: m(X) = 0 und p(X) parallel zum ursprüngliche Photonimpuls.

D.h. es bleibt die Möglichkeit, dass ein Photon in zwei Photonen zerfällt, die sich (mit jeweils halber Frequenz, Energie und Impuls) exakt in Vorwärtsrichtung bewegen.

Diesen Zerfall beobachtet man in der Natur nicht, d.h. das u.g. Feynmandiagramm muss in der QED exakt Null sein. Wie sieht man ein, dass da so ist? Antisymmetrie des dp-Integranden im Fermion-Loop und daher Null?

Powered by phpBB © 2001, 2005 phpBB Group